Anatomy Practice Questions - Exam 1

Lakukan tugas rumah & ujian kamu dengan baik sekarang menggunakan Quizwiz!

2. Cancer spreading via the lymphatics passes into the axillary group of lymph nodes. Which of these axillary groups of nodes is most likely to receive this lymph first? A. Anterior (pectoral) B. Apical (subclavian) C. Central D. Lateral (brachial) E. Posterior (subscapular)

A. Anterior (pectoral) 75% of the lymph from the breast passes to the axillary nodes and, of this group of 20-30 nodes, the anterior (pectoral) group of nodes receives the lymph first (the nodes closest to the axillary tail of the upper outer quadrant of the breast).

A patient is found to have a melanoma (cancer arising in pigment cells) originating in the skin of the left forearm. After removal of the tumor from the forearm, all axillary lymph nodes lateral to the medial edge of the pectoralis minor muscle are removed. Which axillary lymph nodes would not be removed by this procedure? A. Apical B. Central C. Lateral D. Pectoral E. Subscapular

A. Apical If all the lymph nodes lateral to the medial edge of pectoralis minor are removed, the central, lateral, pectoral and subscapular nodes will be removed. The central nodes are found directly under pectoralis minor, while the other three groups of nodes are lateral to the entire muscle. The apical nodes, which are medial to the medial edge of pectoralis minor, will not be removed.

Intravenous fluid administered into the median cubital vein that enters the basilic vein would then most likely empty into which of the following veins? A. Axillary B. Brachial C. Cephalic D. Deep brachial E. Subclavian

A. Axillary The median cubital vein may drain into the basilic vein, which then dives deeply and drains into the axillary vein.

In lymphatic drainage of the breast, the major portion (about 75%) enters eventually into which group of nodes? A. Central axillary B. Deltopectoral C. Lateral axilllary D. Parasternal E. Subscapular

A. Central axillary About 75% of the lymph draining the breast goes to the axillary lymph nodes, via the pectoral lymph nodes. All of this lymph from the pectoral lymph nodes must drain to the central lymph nodes as well. This is why it is so important to examine all these groups of axillary lymph nodes when performing a breast exam. Most of the rest of the lymph drainage from the breast goes to the parasternal nodes, although a small amount goes to the opposite breast and a small amount drains to the abdominal wall.

A wrestler comes off the mat holding his right forearm flexed at the elbow and pronated, with his shoulder medially rotated and displaced inferiorly. Which of his bones is most likely broken? A. Clavicle B. Humerus C. Radius D. Scapula E. Ulna

A. Clavicle Fractures of the clavicle are relatively common and occur most often in the middle third of the bone. The distal fragment is displaced downward by the weight of the shoulder and drawn medially by the action of the pectoralis major, teres major, and latissimus dorsi muscles.

The nerve which passes through the quadrangular space of the posterior shoulder innervates which muscle? A. Deltoid B. Infraspinatus C. Subscapularis D. Supraspinatus E. Teres major

A. Deltoid The quadrangular space is bounded medially by the tendon of the long head of the triceps, laterally by the humerus, superiorly by teres minor, and inferiorly by teres major. The posterior circumflex humeral artery and the axillary nerve traverse this space. The axillary nerve innervates 2 muscles: deltoid and teres minor. So, deltoid is the answer! Infraspinatus and supraspinatus are both innervated by the suprascapular nerve, off the superior trunk of the brachial plexus. Subscapularis is innervated by the upper and lower subscapular nerves, off the posterior cord of the brachial plexus. Teres major is also innervated by the lower subscapular nerve.

A fall on an outstretched hand results in swelling and pain on the lateral aspect of the wrist. Radiographic examination confirms a Colles' fracture. Which of the following bones is most likely fractured? A. Distal radius B. Distal ulna C. Lunate D. Scaphoid E. Trapezium

A. Distal radius The Colles' fracture (a fracture of the distal radius) presents with a classic dinner fork deformity with the dorsal and proximal dis- placement of the distal fragment. This is an extension-compression fracture.

Later, while undergoing physical therapy because of his shoulder injury, he comments that it is very painful when his forearm is brought across his chest (medial or internal rotation of the humerus). You deduce that the pain is due to stretching of the lateral (external) rotators of the shoulder. Which muscle was most likely the source of his pain? A. infraspinatus B. latissimus dorsi C. rhomboideus major D. supraspinatus E. teres major

A. Infraspinatus Infraspinatus and teres minor are the lateral rotators of the shoulder. Infraspinatus is innervated by the suprascapular nerve and teres minor is innervated by the axillary nerve. Latissimus dorsi and teres major are both medial rotators of the arm. Supraspinatus initiates abduction of the arm for approximately the first 15 to 20 degrees--after that, deltoid takes over and allows the arm to continue abducting. Rhombiodeus major and minor are important for retracting the scapula.

An elderly man complained of pain in his shoulder when he brought his forearm and hand behind his back while dressing. It was determined that the pain was caused by stretching of the lateral rotators of his arm during this motion. Which muscle was most likely involved? A. Infraspinatus B. Latissimus dorsi C. Subscapularis C. Supraspinatus D. Teres major

A. Infraspinatus Infraspinatus and teres minor are the two lateral rotators of the arm. These were probably the muscles that this patient had strained. Latissimus dorsi, teres major, subscapularis and pectoralis major are all important medial rotators of the arm. Supraspinatus is the muscle that initiates abduction of the arm through the first 15 degrees.

While observing a mastectomy on a 60-year-old female patient, a medical student was asked by the surgeon to help tie off the arteries that supply the medial side of the breast. The artery that gives origin to these small branches is the: A. Internal thoracic B. Musculophrenic C. Posterior intercostal D. Superior epigastric E. Thoracoacromial

A. Internal thoracic Small branches from the internal thoracic artery, known as medial mammary branches, supply the medial side of the mammary gland. The lateral side of the mammary gland is supplied by the lateral thoracic artery. The musculophrenic artery is a branch of the internal thoracic artery--it travels laterally and supplies blood to the 7th through 9th intercostal spaces. The posterior intercostal arteries are branches from the descending aorta--they supply the lateral and posterior portions of the intercostal space. The superior epigastric artery is a branch of the internal thoracic artery--it supplies the upper rectus abdominis muscle and the upper abdominal wall. The thoracoacromial artery supplies blood to the pectoral muscles, deltoid, subclavius, and the shoulder joint.

During insertion of an IV cannula in the median cubital vein, the patient suddenly lost feeling on the radial side of the forearm. What nerve was injured? A. Lateral antebrachial cutaneous B. Medial antebrachial cutaneous C. Musculocutaneous D. Posterior antebrachial cutaneous E. Superficial radial nerve

A. Lateral antebrachial cutaneous n. There are 3 nerves that might be damaged due to a venipuncture in the median cubital fossa. If the needle goes a bit lateral, the lateral antebrachial cutaneous nerve might be injured. This nerve is a branch of the musculocutaneous nerve which supplies the skin of the lateral side of the forearm. The patient's symptoms (loss of feeling on the radial side of the forearm) match with an injury to the lateral antebrachial cutaneous nerve. If the needle goes a bit medial, it could injure the medial antebrachial cutaneous nerve. This nerve is a direct branch of the medial cord of the brachial plexus--it innervates skin on the medial side of the forearm. If the needle goes too deep, the median nerve might be injured. This would cause the patient to lose sensation on the palmar side of the lateral 3.5 digits. The posterior antebrachial cutaneous nerve is a branch of the radial nerve which supplies the posterior forearm. The superficial radial nerve is a terminal branch of the radial nerve which supplies the dorsum of the hand and the dorsal side of the lateral 2.5 digits. Neither of these nerves would be affected by a venipuncture in the median cubital fossa!

After a penetrating wound in the area of the posterior axillary fold a patient had weakness in extension and adduction of the arm. Which muscle is likely involved? A. Latissimus dorsi B. Pectoralis major C. Levator scapulae D. Rhomboideus major E. Trapezius

A. Lattisimus Dorsi Latissimus dorsi makes the posterior axillary fold, so it's easy to see why that should be the muscle injured following a wound to the posterior axillary fold. Latissimus dorsi is the muscle important for medial rotation, extension, and adduction of the arm; the patient's symptoms fit with an injury to this structure. Pectoralis major medially rotates and flexes the arm--this muscle makes the anterior axillary fold. Levator scapulae elevates the scapula, while rhomboideus major retracts the scapula. Trapezius elevates and depresses the scapula (depending on which part of the muscle contracts), rotates the scapula superiorly, and retracts the scapula. It also helps raise the tip of the shoulder, or acromion.

In a patient with Erb-Duchenne palsy, a nerve arising from the superior trunk of the brachial plexus is nonfunctional. This nerve is the: A. Suprascapular B. Dorsal scapular C. Long thoracic D. Lateral pectoral E. Medial pectoral

A. Suprascapular Remember -- Erb-Duchenne palsy is the avulsion of the C5 and C6 roots of the brachial plexus. The suprascapular nerve comes from the superior trunk of the brachial plexus to innervate supraspinatus and infraspinatus. Since the superior trunk is made entirely from the C5 and C6 nerve roots, it makes sense that the suprascapular nerve would be damaged in Erb-Duchenne palsy. The dorsal scapular nerve comes off of the C5 root to innervate levator scapulae and the rhomboids. This nerve would also be damaged in a case of Erb-Duchenne palsy, but it is not coming off of the superior trunk. (Remember: the question specified the superior trunk!) The long thoracic nerve comes off the roots of C5, C6, and C7 to innervate serratus anterior. This nerve would be affected by Erb-Duchenne palsy, but it's not from the superior trunk. Finally, the lateral and medial pectoral nerves come off the lateral and medial cords of the brachial plexus, respectively. Lateral pectoral nerve would be affected in Erb's palsy, but not medial pectoral.

Would there be loss of cutaneous sensation over the shoulder due to loss of the nerves supplying the C5 dermatome? A. Yes B. No

A. Yes The spinal nerve was severed right by the vertebral column, which means that it had not given off its primary rami that carry cutaneous sensory fibers. So, there would be a loss of cutaneous sensation over the entire C5 dermatome

Elevation of the tip of the patient's right shoulder was still possible indicating that which of the following nerves was intact? A. accessory B. axillary C. dorsal scapular D. suprascapular E. thoracodorsal

A. accessory The accessory nerve (CN XI) is responsible for innervating the trapezius which lifts the tip of the shoulder, or acromion. The axillary nerve, which will be covered with the upper limb, innervates the deltoid muscle. The dorsal scapular nerve innervates rhomboideus major, rhomboideus minor, and levator scapulae. If the dorsal scapular nerve is damaged, the rhomboids will be denervated, and the patient will not be able to retract his scapula fully. The suprascapular nerve, which will also be covered during the upper limb, innervates the supraspinatus and infraspinatus muscles. Finally, the thoracodorsal nerve innervates latissimus dorsi. If this nerve is injured and latissimus dorsi is denervated, the patient will not be able to raise his trunk to his arms (the motion associated with pull-ups). Noticing lots of questions about the accessory nerve, the trapezius, and lifting the acromion? It's important to remember this relationship!

While wandering around in the dark in an unfamiliar home, a visitor accidentally walks into a plate glass door. The door shatters and a shard of glass severs the posterolateral aspect of the woman's neck. Examination reveals that the she is unable to elevate the tip of her shoulder on the injured side. The nerve injured is: A. accessory B. dorsal scapular C. greater occiptal D. spinal nerve C4 E. thoracodorsal

A. accessory The accessory nerve innervates the trapezius which is the muscle responsible for elevating the acromion of the scapula, also known as the tip of the shoulder. If a patient has damaged her accessory nerve, she will be unable to elevate her acromion. The dorsal scapular nerve innervates three muscles: rhomboideus major, rhomboideus minor, and the lower portion of levator scapulae. The rhomboids and trapezius retract the scapula toward the midline. So, if the dorsal scapular nerve is injured and rhomboids are paralyzed, retraction of the scapula on the affected side will be weakened. The greater occipital nerve supplies upper deep neck muscles and cutaneous sensation to the posterior scalp. Spinal nerve C4 refers to the nerve formed by the dorsal and ventral roots of C4. This nerve does not innervate the trapezius. Although branches from the ventral primary rami of C3 and C4 combine with the accessory nerve to form the subtrapezial plexus, C3 and C4 provide only proprioception and are not involved with the motion of the trapezius. Finally, the thoracodorsal nerve innervates latissimus dorsi. The latissimus dorsi is the muscle used to extend the arm or raise the trunk to the arms, as if climbing or doing chin-ups. If the thoracodorsal nerve was injured, a patient would be unable to complete these motions.

A person receives a laceration along the anterior border of the trapezius muscle in the neck and subsequently the point of his shoulder (scapula) sags and he has some difficulty fully abducting his arm. What nerve appears to have been severed? A. accessory (Cr. N. XI) B. axillary C. dorsal scapular D. greater occipital E. suprascapular

A. accessory (CN XI) If the accessory nerve is damaged and the trapezius is denervated, a person will no longer be able to raise the acromion of the shoulder. The dorsal scapular nerve innervates rhomboideus major, rhomboideus minor, and levator scapulae. If the dorsal scapular nerve is damaged, the rhomboids will be denervated, and retraction of the scapula will be weakened. An injury to the greater occipital nerve will result in a loss of sensation on the posterior scalp and denervation of the upper deep neck muscles on the affected side. The axillary nerve and suprascapular nerve will be covered with the upper limb, but for completeness, note that the axillary nerve innervates the deltoid muscle. If this nerve is damaged, the deltoid may atrophy, and the person will be unable to abduct the arm. If the suprascapular nerve is injured, lateral rotation of the humerus will be severely weakened.

After initial examination, the patient is sent to radiology. Radiographs reveal that the portion of the scapula forming the tip or point of the shoulder has been fractured. This bone is the: A. acromion B. angle C. coracoid D. glenoid E. spine

A. acromion The acromion is the point of the scapula that corresponds to the tip of the shoulder. Because most of the scapula is protected by muscles and the thoracic wall, most scapular fractures involve the protruding acromion. The angles of the scapula are inferior to the acromion and not involved with the tip of the shoulder. The coracoid process is a beaklike process on the anterior face of the scapula. It is inferior to the acromion and projects anteriorly. The glenoid cavity is on the lateral side of the scapula. It articulates with the head of the humerus. The spine of the scapula is the ridge of bone on the posterior face of the scapula. Although it continues laterally to become the acromion, it is not the same as the acromion, which specifically refers to the tip of the shoulder.

After initial examination, a patient is sent to radiology. Radiographs reveal that the portion of the scapula forming the tip or point of the shoulder has been fractured. This bone is the A. acromion B. angle C. coracoid D. glenoid E. spine

A. acromion The acromion is the part of the scapula that forms the point of the shoulder--if a patient can't raise this, that means that there is a problem with either trapezius or the accessory nerve! The scapula has two angles--a superior angle, formed by the union of the superior and medial borders and an inferior angle formed by the union of the medial and lateral borders. The coracoid process of the scapula is the point of attachment for pectoralis minor, coracobrachialis, and the short head of the biceps. It is a beak-like anterior projection off the lateral side of the scapula. The glenoid cavity is the articular surface located at the junction of the superior and lateral borders of the scapula-- it's the location where the scapula articulates with the head of the humerus. Finally, the spine of the scapula is a heavy ridge that runs from the medial border of the scapula to the acromion process--it supports the acromion and divides the scapula.

When standing in the Anatomical Position the palms of the hands face: A. anteriorly B. laterally C. medially D. posteriorly E. superiorly

A. anteriorly

The prognosis in breast cancer is poorer as more proximal lymph nodes are found to have cancerous cells in them. Spread of cancer to which of the following axillary nodes would indicate the worst prognosis? A. apical B. central C. lateral D. pectoral E. subscapular

A. apical As lymph drains from the breast into the axillary system, it moves from pectoral, lateral, and subscapular nodes to the central nodes, and then to the apical nodes. If cancer is found in the apical axillary nodes, this is a sign that the cancer has spread through the regional lymphatic system and may have metastasized to the rest of the body.

The surgeon next encounters a large (about 2 cm in diameter) horizontally running vein, fortunately intact, which is also retracted downward. This exposes the damaged artery. He knows that the cords of the brachial plexus are closely associated with this artery, so he carefully retracts one found superolateral and one found inferomedial to the artery and then proceeds to repair the artery itself and close the wound. The large artery that was severed was the: A. axillary B. brachial C. radial D. subscapular E. suprascapular

A. axillary The axillary artery is the continuation of the subclavian artery lateral to the first rib. The cords of the brachial plexus are wrapped around the axillary arteries, so this should be a clue that the axillary artery is the one that was severed. The brachial artery is an extension of the axillary artery distal to teres major. Although these two arteries are continuous, the cords of the brachial plexus are not associated with the brachial artery. The radial artery is a branch of the brachial artery that supplies blood on the radial side of the forearm. The subscapular artery is a branch of the third part of the axillary artery that gives off the thoracodorsal and circumflex scapular arteries. The suprascapular artery is a branch of the thyrocervical trunk that supplies supraspinatus and infraspinatus with blood.

You are in the emergency room when a patient is brought in, the loser in a street fight. He has received a stab wound about 1.5 cm long in the right side of the chest about 1.5 cm below and 1 cm medial to the coracoid process of the scapula. He has lost a lot of bright red blood from a large (approx. 1.2 cm in diameter) severed artery found deep at this location. Intravenous fluids are immediately administered and a surgeon is called in to repair the artery. He begins by making an incision through the skin and subcutaneous tissue just below the clavicle, then cuts the clavicular head of the pectoralis major muscle and retracts it downward to obtain sufficient exposure of the area. He next encounters a partially severed muscle running downward and medially from the coracoid process. He divides the remaining fibers of the muscle and has you retract it downward. This exposes a bloody fat-filled space full of vessels and nerves. Which space or cavity was opened when the surgeon reflected the muscles? A. axillary space B. infraspinatous fossa C. quadrangular space D. subdeltoid bursa E. triangular space

A. axillary space The axilla is a pyramidal space with four walls. The anterior wall is formed by pectoralis major and minor. The posterior wall is formed by the scapula, subscapularis muscle, teres major, and latissimus dorsi. The medial wall is formed by the serratus anterior muscle which is superficial to the ribs. The lateral wall is formed by the intertubercular groove of the humerus. If pectoralis major and minor are reflected, the anterior wall of the axilla is being removed. The infraspinatus fossa is the space inferior to the spine of the scapula, found on the posterior side of the scapula. The quadrangular space is an area bounded medially by the long head of the triceps tendon, laterally by the humerus, superiorly by teres minor, and inferiorly by teres major--the axillary nerve and posterior humeral circumflex artery are transmitted through this space. The subdeltoid bursa is a fluid filled structure that separates the tendon of supraspinatus from the coracoacromial ligament, acromion, and deltoid. The triangular space is bounded laterally by the long head of the triceps, superiorly by teres minor, and inferiorly by teres major--it transmits the circumflex scapular artery.

In the process of escaping from T. rex in Jurassic Park the heroine punctures the skin on the medial side of her wrist on a spiny bush. A few days later, due to the toxin, an infection is seen spreading up the medial side of her arm along the large cutaneous vein extending from the dorsum of her hand to the medial side of her arm. The vein involved is the: A. basilic B. brachial C. cephalic D. median cubital E. ulnar

A. basilic There are two large cutaneous veins running up the forearm. Both veins take origin from the dorsal venous arch of the hand and run up the lateral and medial sides of the forearm. On the medial side (near the 5th digit) there is the basilic vein. On the lateral side, there is the cephalic vein. Since the infection is on the medial side, the correct answer is the basilic vein. (Remember that the hands are supinated in the anatomical position--this comes in handy when you are thinking about the medial and lateral sides of the forearm.) The brachial vein runs with the brachial artery-- it is a deep vein that ends at the level of the elbow. The ulnar vein runs with the ulnar artery, draining the ulnar side of the forearm. Neither of these veins are located in superficial tissue. The median cubital vein is a cutaneous vein, but it is short and only found in the median cubital fossa. It provides a connection between the cephalic vein and basilic vein.

In the axilla the pectoralis minor is a landmark, being closely related to all of the following structures except: A. cephalic vein B. cords of the brachial plexus C. lateral thoracic artery D. medial pectoral nerve E. second part of the axillary artery

A. cephalic vein The cephalic vein is the only structure listed that does not have a special relationship to pectoralis minor. The cords of the brachial plexus are found deep to pectoralis minor. The second part of the axillary artery is defined as the segment of the axillary artery which is covered by the pectoralis minor muscle. So, branches of the second part of the axillary artery, including the lateral thoracic artery, lie deep to pectoralis minor. The medial pectoral nerve pierces pectoralis minor to reach pectoralis major.

Several deficits in muscle function of the right upper limb were noted, including inability to abduct the arm. This was caused by denervation of which muscle? A. deltoid B. infraspinatus C. latissimus dorsi D. teres minor E. trapezius

A. deltoid Although supraspinatus initiates abduction of the arm, the deltoid muscle allows the arm to continue abducting beyond the first 15 to 20 degrees. It is innervated by the axillary nerve, which must have been damaged in this injury. (Remember: the posterior circumflex artery had to be ligated, and it runs in the quadrangular space along with the axillary nerve. So, it's pretty conceivable that the axillary nerve has been injured, too.) Infraspinatus and teres minor are both lateral rotators of the arm. Since teres minor is also innervated by the axillary nerve, it might be injured, too, but the question isn't asking about this muscle. Latissimus dorsi is a medial rotator of the arm, innervated by the thoracodorsal nerve. Finally, trapezius, innervated by the accessory nerve, elevates and depresses the scapula.

An elderly woman falls on her outstretched hand and fractures the surgical neck of her humerus. Several weeks later she presents with significant weakness in abduction of her arm and some weakened extension and flexion. Which of the following nerves is most likely injured? A. Accessory B. Axillary C. Radial D. Subscapular E. Thoracodorsal

B. Axillary Fractures of this portion of the humerus can place the axillary nerve in danger of injury. Her muscle weakness confirms that the deltoid muscle especially is weakened, and the deltoid and teres minor are innervated by the axillary nerve.

3. During a routine physical examination, the physician notes an absent biceps tendon reflex. Which spinal cord level is associated with this tendon reflex? A. C4-C5 B. C5-C6 C. C6-C7 D. C7-C8 E. C8-T1

B. C5-C6 The biceps tendon reflex tests the musculocutaneous nerve and especially the C5-C6 contribution. The triceps tendon reflex tests the C7-C8 spinal contributions of the radial nerve.

A sixteen-year-old boy receives a superficial cut on the thumb side of his forearm. The superficial vein most likely affected is the: A. Basilic B. Cephalic C. Median antebrachial D. Median cubital E. Radial

B. Cephalic There are two large cutaneous veins running up the arm. Both veins take origin from the dorsal venous arch of the hand and run up the lateral and medial sides of the arm. On the medial side (near the 5th digit) there is the basilic vein. On the lateral side (by the thumb), there is the cephalic vein. Since the infection is on the thumb side of the forearm, the correct answer is the cephalic vein. The median antebrachial vein runs down the center of the anterior forearm. The median cubital vein connects the cephalic vein to the basilic vein in the cubital fossa. Finally, the radial vein is a deep vein that runs with the radial artery.

If the second part of the axillary artery was interrupted, collateral blood flow could pass from branches of the thyrocervical trunk into which artery? A. Anterior humeral circumflex B. Circumflex scapular C. Deep brachial D. Posterior humeral circumflex E. Thoracoacromial

B. Circumflex scapular One branch of the thyrocervical trunk is the suprascapular artery. This artery contributes to the scapular anastamosis with the dorsal scapular artery and the circumflex scapular artery. The circumflex scapular artery is a branch of the subscapular artery, which comes from the third part of the axillary artery. So, blood could flow from the suprascapular artery, through the scapular anastamosis, into the circumflex scapular artery, travel retrograde through the circumflex scapular artery and the subscapular artery, and reach the third part of the axillary artery. This would bypass any problems in the second part of the axillary artery. See Netter Plate 410B for a picture of this. The anterior and posterior humeral circumflex arteries are both branches of the third part of the axillary artery. These two arteries anastomose with each other. They would not help shunt blood from the first to the third part of the axillary artery. The deep brachial artery is a branch of the brachial artery found in the deep arm. The thoracoacromial artery is a branch of the second part of the axillary artery. None of these other arteries would help shunt blood over an interruption in the second part of the axillary artery.

An open arterial anastomosis in the shoulder occurs between the suprascapular artery and which other artery? A. Anterior circumflex humeral B. Circumflex scapular C. Dorsal scapular D. Thoracodorsal E. Transverse cervical

B. Circumflex scapular The circumflex scapular artery, the dorsal scapular artery, and the suprascapular artery create arterial anastomoses around the scapula. This means that the scapula will be supplied with blood even if one of these arteries is ligated. Additionally, if the subclavian or axillary artery needs to be ligated, blood can flow from the dorsal scapular artery and suprascapular artery to the circumflex scapular artery. This effectively shunts blood from the first part of the subclavian artery to the third part of the axillary artery so that the upper limb will still receive blood. The connection between the suprascapular and circumflex scapular arteries is termed an open anastomosis because it is grossly visible, compared to the anastomosis with the dorsal scapular, which typically happens within small vessels. The anterior circumflex humeral artery supplies the deltoid muscle. It originates from the third part of the axillary artery and anastomoses with the posterior circumflex humeral artery. The thoracodorsal artery is a branch of the subscapular artery that supplies latissimus dorsi. The transverse cervical artery is a branch of the thyrocervical trunk that occasionally gives rise to the dorsal scapular artery. None of these arteries contribute to the scapular anastomoses.

What muscle is innervated by branches of both the median and ulnar nerves? A. Flexor carpi ulnaris B. Flexor digitorum profundus C. Flexor digitorum superficialis D. Flexor pollicis longus E. Pronator quadratus

B. Flexor digitorum profundus The median and ulnar nerve both innervate flexor digitorum profundus. Flexor carpi ulnaris is innervated by the ulnar nerve only. Flexor digitorum superficialis and flexor pollicis longus are innervated by the median nerve. Pronator quadratus is innervated by the anterior interosseus nerve, which is a branch of the median nerve.

Compression of the median nerve in the carpal tunnel affects which hand muscle(s)? A. Dorsal interossei B. Flexor pollicis brevis C. Flexor pollicis longus D. Opponens digiti minimi E. Palmar interossei

B. Flexor pollicis brevis The recurrent branch of the median nerve innervates the thenar compartment of the hand, including flexor pollicis brevis, abductor pollicis brevis, and opponens pollicis. So, if the median nerve was compressed, all of these muscles might be affected. The dorsal interossei, palmar interossei, and opponens digiti minimi are all muscles of the hand which are innervated by the deep branch of the ulnar nerve. Flexor pollicis longus is innervated by the median nerve, but it is a forearm muscle which is proximal to the carpal tunnel. Therefore, it would not be affected by compressing the median nerve in the carpal tunnel.

Mastitis is a condition which involves: A. A type of leukemia B. Inflammation of the breast C. Infection of lymph nodes D. Mast cells E. Tumors of glandular tissue

B. Inflammation of the breast Mastitis is an inflammation of the breast. It usually occurs during lactation and breast feeding and is usually caused by the organism Staphylococcus aureus. Treating a patient with mastitis that involves infection would include antibiotics, draining an abcess, and excising the diseased mammary duct.

While having an IV needle inserted into the cephalic vein of the forearm, the patient suddenly screamed in pain and felt tingling in part of the skin of the forearm supplied by the nerve accompanying the vein. What nerve was injured? A. Posterior antebrachial cutaneous B. Lateral antebrachial cutaneous C. Medial antebrachial cutaneous D. Musculocutaneous E. Superficial radial

B. Lateral antebrachial cutaneous The lateral and medial antebrachial cutaneous nerves supply the skin of the lateral and medial side of the anterior forearm, respectively. The lateral antebrachial cutaneous nerve is a branch of the musculocutaneous nerve, which runs on the lateral forearm near the cephalic vein. So, this is the nerve that must have been injured. The medial antebrachial cutaneous nerve is a direct branch of the medial cord of the brachial plexus--it runs near the basilic vein. This nerve could be injured during a venipuncture to the basilic vein. The posterior antebrachial cutaneous is a branch of the radial nerve that supplies the skin on the posterior forearm--it is not located near any sites for venipuncture. Finally, the superficial radial nerve supplies cutaneous innervation to the dorsal side of the hand, including the dorsal side of the radial 2 1/2 digits.

The anterior interosseous is a branch of which nerve? A. Axillary B. Median C. Musculocutaneous D. Radial E. Ulnar

B. Median The anterior interosseous nerve is a branch of the median nerve that provides motor innervation to the deep muscles in the flexor compartment, including flexor pollicis longus, the radial half of flexor digitorum profundus, and pronator quadratus. The other related nerve to think about is the posterior interosseous nerve, which is the terminal branch of the deep radial nerve. It provides sensory innervation to the wrist area.

Loss of sensation from the tip of the index finger is indicative of injury to which nerve? A. Radial B. Median C. Ulnar D. Musculocutaneous E. Medial antebrachial cutaneous

B. Median The median nerve provides cutaneous sensation to the palmar face of the radial side of the hand, continuing through the first 3.5 fingers. It also innervates the tips of these fingers and their nail beds on the dorsal side of the hand. So, a lack of sensory innervation at the tip of the index finger means that the median nerve is damaged. The axillary nerve provides cutaneous innervation to the lateral upper arm, and the musculocutaneous nerve provides sensory innervation to the lateral forearm with the lateral antebrachial cutaneous nerve. The radial and ulnar nerve provide sensory innervation to the hand--the radial nerve innervates the radial 2.5 digits on the dorsum of the hand, up to the nail bed. The ulnar nerve innervates the ulnar 2.5 digits on the dorsal side and 1.5 digits on the palmar side of the hand.

When the C5 spinal nerve was severed were any preganglionic sympathetic fibers cut? A. Yes B. No

B. No Preganglionic sympathetic fibers come from the lateral horn of the spinal cord between T1 and L2. They leave the spinal cord through the ventral root before entering the sympathetic chain. Since this injury happened at C5, there are no preganglionic sympathetic fibers coming through the spinal nerve at this level.

You are in the emergency room when a patient is brought in, the loser in a street fight. He has received a stab wound about 1.5 cm long in the right side of the chest about 1.5 cm below and 1 cm medial to the coracoid process of the scapula. He has lost a lot of bright red blood from a large (~1.2 cm in diameter) severed artery found deep at this location. Intravenous fluids are immediately administered and a surgeon is called in to repair the artery. He begins by making an incision through the skin and subcutaneous tissue just below the clavicle, then cuts the clavicular head of the pectoralis major muscle and retracts it downward to obtain sufficient exposure of the area. He next encounters a partially severed muscle running downward and medially from the coracoid process. He divides the remaining fibers of the muscle and has you retract it downward. This exposes a bloody fat-filled space full of vessels and nerves. The muscle running downward and medially from the coracoid process which was partially severed was the: A. Coracobrachialis B. Pectoralis minor C. Long head of the biceps D. Subclavius E. Subscapularis

B. Pectoralis minor Pectoralis minor inserts on the coracoid process of the scapula. Its origin is ribs 3-5, so it runs downward and medially from the coracoid process. Coracobrachialis is a muscle of the upper limb which takes origin from the coracoid process. However, it attaches to the shaft of the humerus and runs laterally. The long head of the biceps, subclavius, and subscapularis are not attached to the coracoid process.

In a fracture of the surgical neck of the humerus, which artery may be injured? A. Subscapular B. Posterior humeral circumflex C. Radial recurrent D. Deep brachial E. Circumflex scapular

B. Posterior humeral circumflex The posterior and anterior circumflex arteries wrap around the humerus near its surgical neck. A fracture to the surgical neck could damage either of these arteries or the axillary nerve. (Remember--the posterior circumflex humeral artery and the axillary nerve cross through the quadrangular space together.) The subscapular artery is a branch of the third part of the axillary artery--it branches to form the thoracodorsal artery and the circumflex scapular artery. The radial recurrent artery is a branch of the radial collateral artery--it contributes to collateral circulation around the elbow. The deep brachial artery is an artery in the deep arm--it is close to the humerus, so fracturing the humerus at mid-arm might result in damage to this vessel.

While riding a bike, a patient fell against a tree and fractured the shaft of the humerus at midlength. What nerve may be injured because of its close proximity to the injury? A. Ulnar B. Radial C. Axillary D. Medial antebrachial cutaneous E. Median

B. Radial The radial nerve and the deep brachial vessels spiral around the shaft of the humerus in the radial groove. If the humerus is fractured, the radial nerve and the deep brachial vessels may be injured, since these structures are closely associated with the shaft of the humerus. The ulnar nerve passes posterior to the medial epicondyle of the humerus--a fracture to this epicondyle may injure the ulnar nerve. The axillary nerve wraps around the surgical head of the humerus-- a fracture near the surgical head may endanger the axillary nerve. The medial antebrachial cutaneous nerve and the median nerve travel superficially with the basilic vein and the brachial artery in a neurovascular bundle. These nerves are not going to be injured by a fracture of the humerus.

In most peripheral nerve injuries, there is usually characteristic sensory loss, producing numbness in a specific area. Which of the following best explains why there is no numbness in this case? A. The motor part of CN XI must have been damaged, while the sensory part remained intact. B. The spinal accessory nerve is unique among peripheral nerves in that it does not carry any sensory fibers whatsoever. C. Sensory loss is not expected from damage of this nerve because no peripheral nerve that innervates a skeletal muscle of the upper limb carries sensory fibers. D. Nerves that innervate skeletal muscles carry only motor neurons, and cutaneous sensory nerves carry only sensory neurons.

B. The spinal accessory nerve is unique among peripheral nerves in that it does not carry any sensory fibers whatsoever. CN XI is truly only motor. Some other motor nerves (e.g., dorsal scapular n.) do not carry any cutaneous sensory neurons, but all nerves to skeletal muscle also carry proprioceptive fibers. These fibers transmit sensory information concerning the state of contraction and position of joints to the central nervous system, allowing for coordination of movement.

A man riding a motorcycle hit a wet spot in the road, lost control, and was thrown from his bike. He landed on the right side of his head and the tip of his shoulder, bending his head sharply to the left and stretching the right side of his neck. Subsequent neurological examination revealed that the roots of the 5th and 6th cervical nerves had been torn away from the spinal cord. Following the above injury, which of the movements of the arm at the shoulder would you expect to be totally lost? A. adduction B. abduction C. flexion D. extension E. medial rotation

B. abduction Injuries to the upper roots of the brachial plexus (C5 and C6) are the most common types of injuries--resulting in a condition known as Erb-Duchenne Palsy. It affects especially the suprascapular, axillary, and musculocutaneous nerves, which causes paralysis of the rotator cuff muscles, biceps, brachialis, coracobrachialis, and deltoid. It also knocks out the upper and lower subscapular nerves, denervating subscapularis and teres major. It knocks out most of the lateral pectoral nerve, but the majority of pect major is innervated by medial pectoral nerve, so it is only weakened. After this injury, the upper limb hangs limply, medially rotated by an unopposed latissimus and pectoralis major muscles, and pronated due to a loss of biceps. So, the limb is constantly adducted and medially rotated. However, the limb can no longer be abducted because both supraspinatus, which initiates abduction, and deltoid, which allows for complete abduction, have been denervated. As far as extension and flexion go: Extension occurs through the actions of the triceps which is innervated by the radial nerve. This nerve should still be intact. Flexion of the arm is not totally lost if biceps brachii and coracobrachialis are denervated, because pectoralis major is not completely lost.

While treating the patient (in the preceding question) the doctor observes, "You were fortunate that the panel did not sever the large artery running down the medial side of your arm." The large artery referred to lies in the neurovascular compartment and supplies most everything below the elbow; this artery is the: A. axillary B. brachial C. deep brachial D. radial E. ulnar

B. brachial The brachial artery runs down the medial side of the arm, near the basilic vein and the median nerve. This artery is a continuation of the axillary artery distal to the teres major. The deep brachial artery is a branch of the brachial artery which runs with the radial nerve in the radial groove of the humerus. The radial and ulnar arteries are branches of the brachial artery which run down the ulnar and radial sides of the forearm.

You are attending an axillary lymph node dissection in a patient with a melanoma in the upper limb. The surgeon says, "We are going to sample the level II lymph nodes posterior to the pectoralis minor muscle." Having excelled in anatomy, you realize that she is referring to the anatomical nodes known as A. apical axillary nodes B. central axillary nodes C. lateral axillary nodes D. lateral pectoral nodes E. subscapular axillary nodes

B. central axillary nodes The central lymph nodes are defined as the group of lymph nodes situated deep to the pectoralis minor at the base of the axilla. The central nodes receive lymph from the lateral, subscapular and pectoral lymph nodes. The lateral nodes are found on the lateral wall of the axilla; the subscapular nodes are found on the posterior wall of the axilla; the pectoral nodes are found on the anterior wall of the axilla. These groups of lymph nodes are lateral to pectoralis minor. The apical lymph nodes are medial to the medial border of the pectoralis minor, so they would not be found under pectoralis minor.

During a fight a man is stabbed in the lateral chest beneath the right arm. The wound does not enter the chest cavity. Physical examination reveals that the vertebral (medial) border of the patient's scapula projects posteriorly and is closer to the midline on the injured side. On return visit the patient complains that he cannot reach as far forward (such as to reach for a door knob) as he could before the injury. The nerve injured which caused these symptoms is the: A. axillary B. long thoracic C. musculocutaneous D. radial E. suprascapular

B. long thoracic Remember--an injury to the long thoracic nerve denervates serratus anterior, meaning that there will be no muscle protracting the scapula and counteracting trapezius and the rhomboids, powerful retractors of the scapula. The long thoracic nerve is derived from the nerve roots of C5-7; this nerve is particularly vulnerable to iatrogenic injury during surgical procedures because it is located on the superficial side of serratus anterior. The axillary nerve innervates teres minor and deltoid. It wraps around the surgical neck of the humerus and is endangered by fractures of the surgical neck. If the axillary nerve was damaged and deltoid was denervated, the patient would be unable to abduct his upper limb beyond 15 to 20 degrees. The musculocutaneous nerve innervates biceps brachi, coracobrachialis, and brachialis. If this nerve was disrupted, the patient would be unable to flex her or his forearm, and have weakened arm flexion. The radial nerve innervates extensors of the forearm and triceps brachi--if this nerve was injured, the patient would no longer be able to extend forearm, but only have slightly weakened arm extension (latissimus is the powerful extensor of the arm). Finally, the suprascapular nerve innervates supraspinatus--the muscle that initiates abduction. Damage to this nerve would prevent the patient from starting to abduct her or his arm.

Neurological testing of a patient reveals no cutaneous sensation on the tip of the index finger. Such a finding would indicate injury to some portion of which nerve? A. axillary B. median C. musculocutaneous D. radial E. ulnar

B. median The median nerve provides cutaneous sensation to the palmar face of the radial side of the hand, continuing through the first 3.5 fingers. It also innervates the tips of these fingers and their nail beds on the dorsal side of the hand. So, a lack of sensory innervation at the tip of the index finger means that the median nerve is damaged. The axillary nerve provides cutaneous innervation to the lateral upper arm, and the musculocutaneous nerve provides sensory innervation to the lateral forearm with the lateral antebrachial cutaneous nerve. The radial and ulnar nerve provide sensory innervation to the hand--the radial nerve innervates the radial 2.5 digits on the dorsum of the hand, up to the nail bed. The ulnar nerve innervates the ulnar 2.5 digits on the dorsal side and 1.5 digits on the palmar side of the hand.

A person riding a mountain bike on a rustic trail hits a rut, the fork of the bike breaks and the person is thrown into a tree, severely fracturing the upper end of his humerus. During the repair the surgeon ties off the artery traveling through the quadrangular space to stop the hemorrhage. Which artery did he ligate? A. dorsal scapular B. posterior circumflex humeral C. scapular circumflex D. subscapular E. suprascapular

B. posterior circumflex humeral The quadrangular space is bounded medially by the long head of the triceps tendon, laterally by the humerus, superiorly by teres minor, and inferiorly by teres major. It is traversed by the axillary nerve and the posterior humeral circumflex artery. The triangular space is medial to the quadrangular space. It is bounded laterally by the tendon of the long head of the triceps, superiorly by teres minor, and inferiorly by teres major. The circumflex scapular artery, which is a branch of the subscapular artery, traverses this space. The dorsal scapular artery and suprascapular artery contribute to the scapular anastamoses with the circumflex scapular artery, but neither of these vessels are in the quadrangular or triangular spaces.

After being thrown from a motorcycle moving at high speed, a 16-year-old female was found to have a paralyzed right pectoralis major muscle. Which set of movements at the shoulder joint would be found greatly weakened? A. Abduction and extension B. Abduction and lateral rotation C. Adduction and flexion D. Lateral rotation and extension

C. Adduction and flexion Pectoralis major flexes, adducts, and medially rotates the arm. It is innervated by the lateral and medial pectoral nerves, from the lateral and medial cords of the brachial plexus.

During the planning of therapeutic intervention for a 54-year-old female patient with cancer of the right breast, a 3rd year medical student would need to first consider where most of the cancer cells would metastasize, which would be: A. Abdominal wall B. Anterior mediastinum C. Axillary lymph nodes D. Opposite breast E. Parasternal lymph nodes

C. Axillary lymph nodes About 75% of the lymph from the breast goes to the axillary lymph nodes via the pectoral lymph nodes. This is the most important place to check for metastasis of the cancer cells! The lymph from the axillary nodes eventually drains into the subclavian lymph trunk. Most of the rest of the lymph drains into the parasternal lymph nodes, while a small amount drains to the abdominal wall and the opposite breast. The anterior mediastinum is not an important place for lymphatic drainage from the breast.

Structures within the carpal tunnel include the: A. Radial bursa B. Ulnar bursa C. Both D. Neither

C. Both The radial bursa and ulnar bursa are both found in the carpal tunnel. These bursae are complex synovial coverings that protect the flexor tendons. The carpal tunnel is formed where the flexor retinaculum spans from the scaphoid and trapezium to the hamate and pisiform, deep and slightly distal to the palmar carpal ligament. This creates a canal that covers the flexor digitorum superficialis tendons, the flexor digitorum profundus tendons, the tendon of flexor pollicis longus, and the median nerve. These tendons in the carpal tunnel are covered by the ulnar and radial bursae. The flexor digitorum superficialis and flexor digitorum profundus tendons are covered by the ulnar bursa, and the tendon of flexor pollicis longus is covered by the radial bursa. So, both bursae are in the carpal tunnel.

Interruption of the median nerve in the cubital fossa affects what movement(s) of the thumb? A. Flexion B. Opposition C. Both D. Neither

C. Both The recurrent branch of the median nerve innervates the thenar compartment of the hand. This nerve innervates opponens pollicis, which opposes the thumb, and flexor pollicis brevis, which helps to flex the thumb. So, disrupting the median nerve would impair both flexion and opposition of the thumb.

During your introductory course to clinical medicine, you are asked to take the radial pulse of your classmate. Which of the following muscle tendons can you use as a guide to locate the radial artery? A. Adductor pollicis longus B. Brachioradialis C. Flexor carpi radialis D. Flexor pollicis longus E. Palmaris longus

C. Flexor carpi radialis The radial pulse can be easily taken at the wrist where the radial artery lies just lateral to the tendon of the flexor carpi radialis muscle.

Breast cancer cells can spread directly to the cranial cavity and brain via the vertebral venous plexus. Through which route can they reach this plexus? A. Axillary lymph nodes B. Internal thoracic vein C. Intercostal veins D. Parasternal lymph nodes E. Thoracoacromial artery

C. Intercostal veins Hematogenous spread through the intercostal veins is the easiest way for breast cancer to reach the internal vertebral venous plexus. It is true that the axillary lymph nodes drain 75% of the lymph from the breast, and the parasternal lymph nodes drain most of the remaining lymph. However, these lymphatic channels are not the major way that cancer would be transmitted to the internal vertebral venous plexus. This plexus of veins would be most likely to receive cancer cells transmitted through the blood. The internal thoracic vein drains some blood from the breast, but it would not direct the blood toward the vertebral column. The thoracromial artery and other arteries do not drain the breast; hence, they would not provide a route for spreading cancer.

In a diving accident that severed the spinal cord below the sixth cervical vertebra, which muscle would be affected? A. Deltoid B. Infraspinatus C. Latissimus dorsi D. Levator scapulae E. Trapezius

C. Latisimus Dorsi If the spinal cord was severed beneath the 6th cervical vertebra, all nerve roots below C6 would be affected. So, latissimus dorsi, which is innervated by the thoracodorsal nerve (C6, C7, C8) would be affected. Deltoid is suplied by the axillary nerve (C5, C6). Infraspinatus is supplied by the suprascapular nerve (C5, C6). Levator scapulae is supplied by the dorsal scapular nerve (C5). Trapezius is supplied by the accessory nerve (CN XI). All of these nerves would be intact, so these muscles would not be affected after the accident.

Which of the following tendons is most vulnerable to inflammation and sepsis in the shoulder joint? A. Glenoid labrum B. Infraspinatus C. Long head of biceps D. Long head of triceps E. Supraspinatus

C. Long head of biceps The long head of the biceps tendon passes through the shoulder joint and attaches to the supraglenoid tubercle of the scapula. An infection in the joint could involve this tendon

During a motorcycle accident, an 18-year-old male landed on the right lateral side of his rib cage with his right upper limb abducted. In the hospital he was found to have "winging" of the right scapula. Which nerve was likely damaged in the accident? A. Accessory B. Lateral pectoral C. Long thoracic D. Phrenic E. Vagus

C. Long thoracic An injury to the long thoracic nerve denervates serratus anterior, meaning that there will be no muscle protracting the scapula and counteracting trapezius and the rhomboids, powerful retractors of the scapula. This means that the scapula will be winged backwards, which is this patient's main symptom. The long thoracic nerve is derived from the nerve roots of C5-7. This nerve is particularly vulnerable to iatrogenic injury during surgical procedures, such as mastectomies, because it is located on the superficial side of serratus anterior. The accessory nerve innervates trapezius--an injury to this nerve might lead to an inability to raise the acromion of the shoulder. The lateral pectoral nerve is a small nerve that provides innervation to pectoralis major. The phrenic nerve innervates the diaphragm. The vagus nerve provides parasympathetic innervation to the thorax and much of the abdominal viscera. The patient's symptoms do not fit with an injury to any of these nerves.

In withdrawing a blood sample from the median cubital vein the needle passes slightly deep and medial; which nerve might possibly be injured? A. Dorsal ulnar cutaneous B. Lateral antebrachial cutaneous C. Medial antebrachial cutaneous D. Posterior antebrachial cutaneous E. Superficial radial

C. Medial antebrachial cutaneous The medial antebrachial cutaneous nerve is a direct branch from the medial cord of the brachial plexus. Since it provides cutaneous sensation to the medial side of the anterior forearm, it is slightly medial to the medial cubital vein and could be injured by a needle. If the needle had gone laterally, it might have injured the lateral antebrachial cutaneous nerve, which is running down the lateral side of the anterior forearm. This nerve is a branch of the musculocutaneous nerve. The posterior antebrachial cutaneous nerve runs on the posterior surface of the arm--it comes from the radial nerve. The dorsal ulnar cutaneous nerve is the nerve which runs on the dorsal side of the hand, providing cutaneous innervation to the ulnar side of the wrist, hand, and the medial 1.5 fingers. Finally, the superficial radial nerve innervates the dorsum of the radial side of the hand.

Because of scarring of a patient's median cubital vein, the technician chooses to insert an infusion needle into her basilic vein at the level of the medial epicondyle. Despite the certainty that the needle does not pass through the deep (investing) fascia, there is still a chance that it might nick or impale which of the following? A. Brachial artery B. Lateral antebrachial cutaneous nerve C. Medial antebrachial cutaneous nerve D. Median nerve E. Radial nerve

C. Medial antebrachial cutaneous nerve The basilic vein is on the medial side of the arm, so a venipuncture into the basilic vein might damage the medial antebrachial cutaneous nerve which also runs on the medial side of the arm. The brachial artery, median nerve, and radial nerve are deeper structures that would not be damaged during a venipuncture. The lateral antebrachial cutaneous nerve is near the cephalic vein, not the basilic vein.

During an industrial accident, a sheet metal worker lacerates the anterior surface of his wrist at the junction of his wrist and hand. Examination reveals no loss of hand function, but the skin on the thumb side of his palm is numb. Branches of which nerve must have been severed? A. Lateral antebrachial cutaneous B. Medial antebrachial cutaneous C. Median D. Radial E. Ulnar

C. Median The median nerve provides sensory innervation to the skin of the radial 3.5 fingers of the palm. So, the patient's loss of cutaneous sensation is suggestive of a median nerve injury. The location of the injury also implies that there has been an injury to the median nerve--this nerve enters the hand by crossing over the anterior side of the wrist. The lateral and medial antebrachial cutaneous nerves provide cutaneous innervation to the anterior side of the forearm--the symptoms here are not consistent with an injury to these nerves. The radial nerve innervates the radial side of the dorsum of the hand but does not innervate the palmar side of the hand. The ulnar nerve innervates the medial (ulnar) side of both the dorsum and palm of the hand.

The lateral antebrachial cutaneous nerve comes from the: A. Axillary nerve B. Medial cord nerve C. Musculocutaneous nerve D. Radial nerve E. Ulnar nerve

C. Musculocutaneous n. The musculocutaneous nerve provides cutaneous innervation to the skin of the anterolateral side of the forearm through the lateral antebrachial cutaneous nerve. The axillary nerve supplies the skin of the upper lateral arm with the superior lateral brachial cutaneous nerve. The radial nerve supplies cutaneous innervation to the skin of the posterior arm, forearm, and hand through many different cutaneous nerves. The ulnar nerve supplies sensory innervation to the skin of the medial side of the wrist and hand and skin of the medial 1 1/2 digits on the palmar side, and 2 1/2 digits on the dorsum of the hand. If you are having problems conceptualizing these areas of cutaneous innervation, check out on-line color pictures in the dissector answers, or plate 481 in Netter's!

If the right dorsal scapular nerve were cut near its origin, what would result: A. Skin of the upper back on the right side would be numb B. The point of the right shoulder would droop C. Scapular retraction on the right would be weakened D. Extension of the right arm would be weakened E. Inability to adduct the right arm

C. Scapular retraction on the right would be weakened The dorsal scapular nerve is a motor nerve off the C5 nerve root that innervates the rhomboids and levator scapula. These muscles help to retract and elevate the scapula, so these motions would be weakened following that damage. The skin of the upper back on the right side is innervated by the dorsal primary rami of a spinal nerve. The point of the right shoulder, the acromion, is elevated by trapezius. Trapezius is innervated by the accessory nerve, so the point of the shoulder would droop if the accessory nerve was damaged. Latissimus dorsi, innervated by the thoracodorsal nerve, allows for extension and adduction of the arm.

The long thoracic nerve innervates which muscle? A. Anterior scalene B. Middle scalene C. Serratus anterior D. Teres major E. Subscapularis

C. Serratus anterior Serratus anterior is innervated by the long thoracic nerve. Serratus anterior keeps the scapula held forward, balancing trapezius and the rhomboids which retract the scapula. If the long thoracic nerve is injured (which is common in surgery, since the long thoracic nerve is on the superficial side of serratus anterior), you may see a "winged scapula" protruding posteriorly. The anterior scalene muscle is innervated by C5-C7, and the middle scalene is innervated by C3-C8. Teres major is innervated by the lower subscapular nerve from the posterior cord of the brachial plexus. Subscapularis is innervated by the upper and lower subscapular nerves from the posterior cord of the brachial plexus.

Which muscle is the strongest medial rotator of the arm? A. Coracobrachialis B. Infraspinatus C. Subscapularis D. Supraspinatus E. Teres minor

C. Subscapularis Subscapularis is the only muscle listed that is a strong medial rotator. There are four muscles which are strong medial rotators of the arm: subscapularis, pectoralis major, teres major, and latissimus dorsi. Two of the muscles mentioned are the lateral rotators of the arm: teres minor and infraspinatus. Coracobrachialis flexes and adducts the arm. Supraspinatus is an important muscle for initiating abduction of the arm through the first 15 degrees.

In a case of Erb's palsy, where roots C5 and C6 of the brachial plexus are avulsed (torn out) which muscle is paralyzed? A. Latissimus dorsi B. Pectoralis minor C. Supraspinatus D. Trapezius E. Triceps brachii

C. Supraspinatus The C5 and C6 roots make the superior trunk of the brachial plexus. So, all the muscles innervated by nerves from the superior trunk of the brachial plexus will be denervated following the avulsion. Supraspinatus is innervated by the suprascapular nerve, which comes off of the superior trunk of the brachial plexus. This means that avulsing the C5 and C6 nerve roots would denervate supraspinatus and paralyze that muscle. Latissimus dorsi is innervated by the thoracodorsal nerve, which comes off the posterior cord of the brachial plexus. The thoracodorsal nerve is made of contributions from C6, 7, and 8, so this nerve would be mostly intact. Pectoralis minor is innervated by the medial pectoral nerve, off the medial cord of the brachial plexus. It is made of contributions from C8 and T1. Trapezius is innervated by the accessory nerve -- CN XI. This means that trapezius is not innervated by the brachial plexus. Finally, triceps brachii is innervated by the radial nerve. The radial nerve is a terminal branch of the posterior cord of the brachial plexus, and it is made of contributions from C5, 6, 7, 8, and T1. Triceps brachi would not be entirely paralyzed following an injury to the C5 and C6 nerve roots.

A baseball pitcher delivers a 97-mph fastball to a batter and suddenly feels a sharp pain in his shoulder on release of the ball. The trainer examines the shoulder and concludes that the pitcher has a rotator cuff injury. Which muscle is most vulnerable and most likely torn by this type of injury? A. Infraspinatus B. Subscapularis C. Supraspinatus D. Teres major E. Teres minor

C. Supraspinatus The supraspinatus muscle is most often torn in rotator cuff injuries. Repeated abduction and flexion can cause the tendon to rub on the acromion and coraco-acromial ligament, leading to tears or rupture.

After trying to throw a curve ball, a pitcher lost sensation from the tip of the little finger. This indicates injury to which nerve? A. Radial B. Median C. Ulnar D. Musculocutaneous E. Medial antebrachial cutaneous

C. Ulnar The ulnar nerve innervates the medial 1.5 digits on the palmar surface of the hand, and 2.5 digits on the dorsal side. So, this is the nerve responsible for innervating the tip of the little finger. The radial nerve innervates the dorsal side of the lateral 2.5 digits, but does not innervate the tips of these fingers. The median nerve, which innervates the palmar side of the lateral 3.5 digits, also innervates the fingertips of these 3.5 fingers. The musculocutaneous nerve does not provide cutaneous innervation to the skin, but its branch, the lateral antebrachial cutaneous nerve, innervates the lateral skin of the forearm. The medial antebrachial cutaneous nerve innervates the medial skin of the forearm - this nerve is a direct branch of the medial cord of the brachial plexus.

The cords of the brachial plexus are: A. above the clavicle, medial to the scalenus anticus (anterior scalene). B. above the clavicle, behind the scalenus anticus (anterior scalene). C. at or below the clavicle, closely related to the axillary artery. D. at or below the clavicle, closely related to the axillary vein.

C. at or below the clavicle, closely related to the axillary artery. The cords of the brachial plexus are closely related to the axillary artery, at or below the level of the clavicle. You should have seen this in the dissection--the cords were wrapped around the axillary artery. The axillary vein is anterior to the axillary artery and is not associated with the cords of the brachial plexus. As far as the scalene muscles go, you'll learn more about this in the head and neck, but know that the roots, not the cords, of the brachial plexus are the structures found between the anterior and middle scalene muscles.

In old age, the supraspinatus tendon is sometimes ruptured where it blends with the capsule of the shoulder joint. Following this kind of injury one might expect A. difficulty in adducting the arm B. difficulty in flexing the arm C. difficulty in abducting the arm D. difficulty in extending the arm E. difficulty in medially rotating the arm

C. difficulty in abducting the arm Supraspinatus is one of the four muscles that comprises the rotator cuff, along with infraspinatus, teres minor, and subscapularis. Supraspinatus is an essential muscle for initiating abduction of the arm, so if the supraspinatus tendon is injured, it will be difficult for a patient to begin abducting the arm. After the arm is abducted about 15 degrees, the deltoid will take over and continue abducting the arm. Teres major is the muscle which adducts the arm. Biceps brachii and coracobrachialis flex the arm. The long head of the triceps brachii extends the arm. Finally, pectoralis major, teres major, subscapularis, and latissimus dorsi all medially rotate the arm.

In the process of doing an axillary lymph node dissection in a 50 year-old patient, the surgery resident cleans the space between the pectoralis major and minor muscles, in an attempt to remove all of the lateral pectoral lymph nodes. Upon recovery it is noted that the patient's lower pectoralis major is paralyzed. The nerve most likely injured is the: A. axillary B. lateral pectoral C. medial pectoral D. suprascapular E. thoracodorsal

C. medial pectoral Since the medial pectoral nerve pierces pectoralis minor to reach pectoralis major, it seems likely for this nerve to be injured from trauma to the space between pectoralis minor and major. Also, remember that the medial pectoral nerve innervates the inferior part of pectoralis major, while the lateral pectoral nerve innervates the superior part of pectoralis major. The medial pectoral nerve innervates both pectoralis major and pectoralis minor, while the lateral pectoral nerve innervates pectoralis major only. These two nerves are named after their origin from two different cords of the brachial plexus. (This explains why their names and relative locations are reversed from what you might expect.) The thoracodorsal nerve (which is also derived from the brachial plexus) innervates latissimus dorsi, and is not involved with the pectoral muscles. The other two nerves, axillary and suprascapular, are also derived from the brachial plexus and will be studied along with the upper limb.

As an inexperienced phlebotomist (blood drawer) attempts to insert the needle to draw blood from the median cubital vein, the patient suddenly screams and complains of pain and burning in the middle and thumb side of his palm. The nerve accidentally impaled on the needle was the A. lateral antebrachial cutaneous B. medial antebrachial cutaneous C. median D. ulnar E. posterior antebrachial cutaneous F. radial G. superficial radial

C. median The median nerve supplies cutaneous sensation to the skin of the radial half of the palm and the palmar side of the lateral 3 1/2 digits. The symptoms reported by the patient match with this area of cutaneous innervation. Additionally, the median nerve lies in the cubital fossa, deep to superficial veins like the median cubital vein. So, it seems reasonable that this was the injured nerve. The lateral and medial antebrachial cutaneous nerves supply the skin of the lateral and medial side of the anterior forearm, respectively. The lateral antebrachial cutaneous nerve is a branch of the musculocutaneous nerve, while the medial antebrachial cutaneous nerve is a direct branch of the medial cord of the brachial plexus. The posterior antebrachial cutaneous is a branch of the radial nerve that supplies the skin on the posterior forearm. The ulnar nerve provides cutaneous inervation to the skin of the medial side of the wrist and hand, including the skin of the medial 1 1/2 digits. the ulnar nerve supplies both the palmar and dorsal side of this region. Finally, the superficial radial nerve supplies cutaneous innervation to the dorsal side of the hand, including the dorsal side of the radial 3 1/2 digits.

While putting metal panels on the roof of a barn, one of the panels slips out of the hands of the man on the roof. During an attempt to catch the panel, a worker below is struck by its sharp edge. The panel hits across the anterior surface of his right arm at midlength and the impact severs all of the tissue to the bone. When examined in the emergency room it is noted that the patient can only weakly flex his elbow and the lateral side of his forearm is numb. In addition to the muscles, which nerve is injured? A. axillary B. median C. musculocutaneous D. radial E. ulnar

C. musculocutaneous The musculocutaneous nerve innervates biceps brachii and coracobrachialis--muscles which flex the arm. Since this man cannot flex his arm, it appears that the musculocutaneous nerve has been damaged. He had also lost sensation over the lateral part of his forearm, indicating that the lateral antebrachial cutaneous nerve has been damaged. This nerve is a branch of the musculocutaneous nerve. The axillary nerve innervates deltoid and teres minor--an injury to this nerve would prevent the patient from abducting his arm. The median nerve innervates the muscles that permit flexion at the wrist and some muscles in the hand, especially of the thumb. The radial nerve innervates the extensors of the arm, elbow, wrist, and hand. The ulnar nerve innervates muscles of the hand, primarily. The listed symptoms do not match injuries to any of these other nerves.

After a jarring blow to the left anterior shoulder region, a young field hockey player was told by an examining physician that she had a muscle tear that resulted directly from the superolateral distraction of a fractured coracoid process. Which muscle was torn? A. Deltoid B. Pectoralis major C. Pectoralis minor D. Serratus anterior E. Subclavius

C. pectoralis minor Of the muscles listed, pectoralis minor is the only one which is attached to the coracoid process. The deltoid originates from the clavicle, acromion and scapular spine and inserts on the deltoid process of the humerus. Pectoralis major originates from the clavicle, sternum, and ribs and inserts on the crest of the greater tubercle of the humerus. Serratus anterior originates on the ribs and inserts on the medial border of the costal surface of the scapula. Subclavius originates on the first rib and inserts on the clavicle. So, none of these other muscles would be detached by a fracture of the coracoid process. What other muscles are attached to the coracoid process? Coracobrachialis and the short head of the biceps.

Later when you see the patient it is clear that he has pronounced neurological deficits. There is lack of cutaneous sensation between his thumb and index finger on the dorsal side, diminished sensation on the dorsal surface of the forearm, arm, and lateral side of the shoulder. He has "wrist drop" (inability to extend the wrist), lack of extension at the elbow, and although abduction can be initiated, it cannot be completed. Flexion, extension, medial and lateral rotation of the arm are not significantly affected. There is no "winging of the scapula." Later exploration by a neurosurgeon reveals that a single large nerve bundle posterior to the artery was also severed, accounting for the neurological findings The large nerve bundle lying posterior to the artery which was also severed was the: A. C 7 root B. middle trunk C. posterior cord D. posterior division E. radial nerve

C. posterior cord Start this question by thinking through the clinical findings -- the dorsal side of the hand between the first and second fingers, and the dorsal side of the forearm, arm, and lateral side of the shoulder are innervated by cutaneous branches of the radial nerve. So, the loss of cutaneous sensation is consistent with an injury to the radial nerve. The difficulty extending at the wrist and elbow is also consistent with a radial nerve injury (remember--the radial nerve innervates the extensors). Since the patient can initiate abduction, the suprascapular nerve is intact, but there must be an injury to the axillary nerve since deltoid cannot complete abduction. This combination of injuries to the axillary nerve and the radial nerve point to damage to the posterior cord of the brachial plexus. An injury to the C7 nerve root or the middle trunk would result in a more diffuse injury. Since C7 creates the middle trunk and the middle trunk contributes to the lateral and posterior cords, an injury to C7 would impair the branches from all these structures. Injury to a posterior division of one of the trunks might cause a deficit in the posterior cord, but this is not the clearest answer. Finally, an injury to the radial nerve would not explain the symptoms associated with the loss of the axillary nerve--the only scenario that makes sense here is an injury to the posterior cord of the brachial plexus.

The patient was unable to retract his scapula because of paralysis of the trapezius and A. infraspinatus B. levator scapulae C. rhomboids D. serratus posterior superior E. supraspinatus

C. rhomboids The rhomboids retract the scapula. They are innervated by the dorsal scapular nerve, which comes from C5. So, they will be denervated following this injury to the C5 nerve roots. Infraspinatus is important for laterally rotating the arm, and levator scapulae raises the scapula but does not retract it. Serratus posterior superior elevates the upper ribs and supraspinatus initiates abduction of the arm.

During a strenuous game of tennis a 55 year old woman complained of severe shoulder pain that forced her to quit the game. During physical examination it was found that she could not initiate abduction of her arm, but if her arm was elevated to 45 degrees from the vertical (at her side) position, she had no trouble fully abducting it. Injury to which muscle was responsible? A. deltoid B. infraspinatus C. supraspinatus D. teres major E. trapezius

C. supraspinatus Supraspinatus is responsible for initiating abduction of the arm, while deltoid is responsible for continuing abduction of the arm past the first 15 or 20 degrees. Since this patient can abduct her arm when it is lifted to 45 degrees, deltoid seems to be intact. But, she can't initiate the motion, so you know that she has probably injured supraspinatus. Infraspinatus rotates the arm laterally, and teres major rotates the arm medially. Trapezius elevates and depresses the scapula--a problem with this muscle is most evident if a patient has trouble raising the acromion of her shoulder.

Following the above injury there would most likely be diminished cutaneous sensation over what part of the upper limb? A. the back of the shoulder B. the pectoral region C. the top of the shoulder and the lateral side of the arm D. the medial side of the arm and forearm E. the tip of the little finger

C. the top of the shoulder and the lateral side of the arm The C5 and C6 dermatomes cover the top of the shoulder and lateral side of the arm. The T1 and C8 dermatomes cover the medial side of the arm, with C8 extending to the tip of the little finger. The back of the shoulder is covered by numerous dermatomes, including C6, C7, C8, and T1. Finally, the pectoral region is covered by T1, T2, and T3 dermatomes. To visualize this, see Netter Plate 465.

A 33-year-old woman undergoes a lymph node biopsy of her deep cervical nodes on the left side of her neck. Immediately following surgery, she complains of weakness in her left shoulder. On exam, the left shoulder droops, and she is unable to raise the point of her shoulder. She denies numbness in her shoulder, back, and neck. What nerve appears to have been inadvertently cut during the biopsy? A. Greater occipital n. B. Spinal n. C3 C. Dorsal scapular n. D. Accessory n. (Cranial Nerve XI) E. Cutaneous nn. of the back (dorsal primary rami)

D. Accessory n (CN XI) The accessory nerve innervates the trapezius m., the only muscle that can elevate the tip of the shoulder (the acromion). Thus, it appears that this patient's accessory nerve (CN XI) was severed or injured during the biopsy.

In a fracture of the midshaft of the humerus, which artery is most likely to be injured? A. Subscapular B. Posterior circumflex humeral C. Radial recurrent D. Deep brachial E. Circumflex scapular

D. Deep brachial The deep brachial artery is an artery in the deep arm, wrapping around the humerus in the radial groove. It is close to the humerus, so fracturing the humerus at mid-arm might result in damage to this vessel or the radial nerve. The subscapular artery is a branch of the third part of the axillary artery--it branches to form the thoracodorsal artery and the circumflex scapular artery. The posterior and anterior circumflex arteries wrap around the humerus near its surgical neck. A fracture to the surgical neck could damage either of these arteries or the axillary nerve. The radial recurrent artery is a branch of the radial collateral artery--it contributes to anastomoses around the elbow. The circumflex scapular artery is a branch of the subscapular artery. It travels to the posterior scapula to contribute to the scapular anastomosis.

Which of the following muscle-nerve combinations is tested when spreading the fingers against resistance? A. Abductor digiti minimi-median B. Abductor pollicis brevis-radial C. Abductor pollicis longus-median D. Dorsal interossei-ulnar E. Palmar interossei-ulnar

D. Dorsal interossei-ulnar The dorsal interossei are innervated by the ulnar nerve and abduct the fingers (the little finger and thumb have their own abductors). This action is easily tested in a patient; dorsal interossei abduct the fingers (DAB) and the palmar interossei adduct the fingers (PAD).

The vein of choice for withdrawing blood is the: A. Basilic B. Cephalic C. Median antebrachial D. Median cubital

D. Median cubital The median cubital vein connects the cephalic and basilic veins in the cubital fossa. This vein shunts blood from the cephalic vein to the basilic vein. Venipunctures are usually done in the median cubital vein, so this is the best answer to pick. Another reason that median cubital vein is a favorite is the fact that it is anchored in place by a perforating vein connecting to the brachial veins - so that it doesn't move out of the way of the venipuncture needle. However, don't forget that venipunctures can be done in other veins, including the basilic and cephalic veins. Both of these veins arise from the dorsal venous arch of the hand--the basilic vein travels up the medial side of the arm and the cephalic vein travels up the lateral side of the arm. The median antebrachial vein travels in the center of the forearm and drains into the median cubital vein.

A patient with a midshaft compound humeral fracture presents with bleeding and clinical signs of nerve entrapment. Which of the following nerves is most likely injured by the fracture? A. Axillary B. Median C. Musculocutaneous D. Radial E. Ulnar

D. Radial The radial nerve spirals around the posterior aspect of the midhumeral shaft and can be stretched or contused by a compound fracture of the humerus. This nerve innervates all the extensor muscles of the upper limb (posterior compartments of the arm and forearm).

A woman riding a mountain bike on a rough trail hits a rut and is thrown from the bike. Her upper arm hits a tree, fracturing the humerus just above the insertion of the teres major muscle. Later, during examination, it is noted that she cannot extend her forearm at the elbow or hand at the wrist. Which nerve must have been injured? A. Axillary B. Median C. Musculocutaneous D. Radial E. Ulnar

D. Radial The radial nerve spirals around the posterior surface of the shaft of the humerus in the radial groove. It is endangered by humeral fractures distal to the surgical head of the humerus, which is a pretty good description of this patient's injury. The loss of extension at the elbow and wrist also implies damage to the radial nerve--remember, the radial nerve innervates the extensors! The axillary nerve is damaged by humeral fractures at the surgical neck--an injury to this nerve would lead to an inability to abduct the arm. The median nerve is located near the distal end of the humerus--if this nerve was injured in an accident, a patient might have trouble flexing the wrist. The ulnar nerve is located near the medial epicondyle of the humerus--an injury to this nerve would lead to muscular deficits in the hand. Finally, the musculocutaneous nerve is not usually damaged with humeral fractures.

A patient is severely limited in extension at the wrist joint after several months in a cast following a Colles fracture. Which joint would be especially important in therapy to regain full extension? A. carpometacarpal B. distal radioulnar C. midcarpal D. radiocarpal E. ulnocarpal

D. Radiocarpal The radiocarpal joint is the joint commonly known as the wrist joint--it is a condyloid (oval) type of synovial joint that allows for flexion and extension, abduction and adduction, and circumduction. A Colles fracture is a fracture of the distal end of the radius--this is why this sort of break would limit movement between the radius and carpals. The carpometacarpal joint is found between the distal row of carpals and the metacarpals--these joints are mobile for the thumb and little finger, allowing extension, flexion, abduction, and adduction. However, the carpometacarpal joints are quite immobile for the middle three fingers. The distal radioulnar joint is located between the distal ends of the radius and ulna--this joint allows the radius and ulna to rotate around each other during pronation and supination. The midcarpal joint is located between the proximal and distal row of carpals--this joint is important for flexion and extension of the hand. As for the "ulnocarpal joint," the ulna does not articulate with the carpal bones--it articulates with the distal end of the radius only.

Postoperative examination revealed that the medial border and inferior angle of the left scapula became unusually prominent (projected posteriorly) when the arm was carried forward in the sagittal plane, especially if the patient pushed with outstretched arm against heavy resistance (e.g., a wall). What muscle must have been denervated during the axillary dissection? A. Levator scapulae B. Pectoralis major C. Rhomboideus major D. Serratus anterior E. Subscapularis

D. Serratus anterior Serratus anterior, innervated by the long thoracic nerve, draws the scapula forward. If it is denervated, there is no muscle to oppose the motion of the trapezius which is elevating and retracting the scapula. The medial border of the scapula falls away from the posterior chest wall and begins to look like an angel's wing. This is termed a "winged scapula." A winged scapula commonly occurs after an injury to the long thoracic nerve, which runs on the superficial surface of serratus anterior and is particularly vulnerable to trauma. The long thoracic nerve contains contributions from C5, 6, and 7, so remember the saying "C5, 6, and 7 keep the wings from heaven." This is a classic scenario to remember!

Since the spinal accessory nerve carries no proprioceptive fibers, which of the following best explains the ability to execute coordinated movement of the trapezius muscle? A. Because the trapezius m. is the only muscle that inserts on the acromion, allowing for elevation of the shoulder, the central nervous system does not need proprioceptive feedback for coordinated movement of that muscle. B. Proprioceptive fibers from nerves innervating other muscles that also insert on the clavicle and scapula, including levator scapulae m., rhomboid major m., rhomboid minor m., and sublavius m., provide enough sensory information to the central nervous system that coordinated movement of the trapezius m. can occur without proprioceptive information from that muscle. C. The greater occipital n., a branch of C2, sends proprioceptive fibers from the nuchal ligament and the trapezius m. to the central nervous system to allow for coordinated movement. D. Spinal nerves C3 and C4 carrying proprioceptive fibers combine with branches of the spinal accessory n. in the subtrapezial plexus before innervating the trapezius m.

D. Spinal nerves C3 and C4 carrying proprioceptive fibers combine with branches of the spinal accessory n. in the subtrapezial plexus before innervating the trapezius m. Proprioceptive information from the trapezius m. is transmitted along spinal nerves C3 and C4 back to the cervical spinal cord, from whence the accessory nerve originates. (Its path from the cervical spinal cord up to the jugular foramen from which it begins its final journey toward its targets will be further explored during the Neuro sequence.) The accessory n. also innervates the sternocleidomastoid m., proprioceptive fibers for which are provided by spinal nerves C2 and C3.

Following a gunshot wound to the shoulder, a patient is unable to abduct the arm between 0 and 15 degrees, with reduced lateral rotation of the humerus. What nerve is probably injured? A. Accessory (CN XI) B. Axillary C. Median D. Suprascapular E. Ulnar

D. Suprascapular Since the patient cannot initiate abduction of the arm, you should know that the supraspinatus muscle is injured or denervated. This is the muscle that initiates abduction of the arm through the first 15 degrees. (Deltoid, innervated by the axillary nerve, continues abducting the arm after the first 15 degrees of motion.) The suprascapular nerve innervates supraspinatus, so there must be some damage to that nerve. The suprascapular nerve also innervates infraspinatus, a lateral rotator of the arm. So, the inability to initiate abduction plus the weakness in lateral rotation all point to an injury to the suprascapular nerve! The accessory nerve innervates trapezius and sternocleidomastoid. An injury to this nerve would be most noticible if a patient could not raise the tip of the shoulder! The axillary nerve innervates teres minor and deltoid. A patient with an injury to this nerve would present with an inability to abduct the arm after the first 15 degrees of motion, along with some weakened lateral rotation. The median nerve innervates muscles which flex the wrist as well as the muscles of the thumb. An injury to this nerve would impair flexion of the wrist and thumb and an inability to oppose the thumb. The ulnar nerve innervates most of the muscle of the hand. An injury to this nerve would result in weakened hand motions.

The clavipectoral fascia is penetrated by which artery? A. Anterior circumflex humeral B. Axillary C. Subscapular D. Thoracoacromial E. Thoracodorsal

D. Thoracoacromial The thoracoacromial artery pierces the clavipectoral fascia before giving off its four branches: pectoral, clavicular, deltoid, and acromial. It supplies pectoralis major, pectoralis minor, the deltoid muscle, and the acromioclavicular joint. It is a branch off of the axillary artery. The axillary artery and all its other branches, including the anterior circumflex humoral and subscapular arteries, run deep to the clavipectoral fascia. The thoracodorsal artery is a branch of the subscapular artery which also runs deep to the fascia.

The transverse cervical artery is severed in an auto accident. What muscle would be affected the most? A. Levator scapulae B. Rhomboideus minor C. Rhomboideus major D. Trapezius E. Latissimus dorsi

D. Trapezius The transverse cervical artery supplies blood to trapezius. Levator scapulae and the rhomboids receive blood from the dorsal scapular artery. Latissimus dorsi receives blood from the thoracodorsal artery.

Due to the bleeding, the surgeon ligated (tied off) the suprascapular artery as it crossed the top of the scapula. There was no concern however, as she knew that direct collateral branches of which artery would supply the normal distribution of the suprascapular artery. A. anterior circumflex humeral B. posterior circumflex humeral C. scapular circumflex D. subscapular E. thoracodorsal

D. scapular circumflex The scapular circumflex artery, which is a branch of the subscapular artery, contributes to a scapular anastamosis with the suprascapular artery and the dorsal scapular artery. So, if one of these arteries needs to be ligated, the other ones will supply the scapula. The anterior and posterior circumflex humeral arteries anastomose with each other, but they do not contribute to the blood supply of the scapula. The thoracodorsal artery is another branch of the subscapular artery (just like the scapular circumflex artery). It supplies latissimus dorsi.

While you are stitching up his hand, he notes that you did not have to give him an anesthetic since the area between his thumb and index finger on the dorsal side was already numb. Which nerve must have been injured (most likely by the fracture of his wrist) for this area to be numb? A. lateral antebrachial cutaneous B. medial antebrachial cutaneous C. median D. superficial radial E. superficial ulnar

D. superficial radial The superficial radial nerve provides cutaneous innervation to the radial side of the dorsum of the hand for the first 2 1/2 digits. An injury to this nerve would correlate to the loss of sensation between the thumb and index finger on the dorsum of the hand. The lateral antebrachial cutaneous innervates the lateral anterior side of the forearm--it is a branch of the musculocutaneous nerve. The medial antebrachial cutaneous nerve comes off the medial cord of the brachial plexus--it innervates the medial anterior side of the forearm. The median nerve provides cutaneous branches that innervate the radial side of the palmar or volar surface of the hand for the first 3 1/2 digits. Finally, the superficial ulnar nerve innervates the ulnar side of the hand on both the palm and the dorsum, covering the final 1 1/2 fingers on the volar surface and 2 1/2 fingers on the dorsum.

An elderly patient complains of shoulder pain and has difficulty abducting his arm. Arthroscopy is done in which a dye is injected into the shoulder joint and an X-ray taken. The radiologist notes that the dye has leaked from the shoulder joint into the subacromial bursa. What tendon would need to be ruptured for this to occur? A. deltoid B. infraspinatus C. latissimus dorsi D. supraspinatus E. teres minor

D. supraspinatus The shoulder joint is separated from the subacromial bursa by the supraspinatus tendon, which is inserting on the greater tubercle of the humerus. So, if the supraspinatus tendon is ruptured, dye injected into the shoulder joint will not just stay in the shoulder joint--instead, it will diffuse and leak into the area around the joint, including the subacromial bursa. Deltoid is a much more superficial muscle--it is not related to the shoulder joint capsule. Infraspinatus and teres minor also insert on the greater tubercle of the humerus, but they insert a bit inferior to supraspinatus, so they are not related to the subacromial bursa. Latissimus dorsi is not part of the rotator cuff--it inserts on the floor of the intertubercular groove of the humerus.

The rotator cuff is composed of all of the following muscles except: A. infraspinatus B. subscapularis C. supraspinatus D. teres major E. teres minor

D. teres major Teres major is not part of the rotator cuff. It does not insert on the greater or lesser tubercle of the humerus--instead, it inserts on the crest of the lesser tubercle of the humerus, which is distal to the lesser tubercle. It is innervated by the lower subscapular nerve and it is a medial rotator of the arm. The other four muscles comprise the rotator cuff--they all insert on the greater or lesser tubercles of the humerus. Supraspinatus inserts into the upper facet of the greater tubercle of the humerus and into the capsule of the shoulder joint. Infraspinatus inserts into the middle facet of the greater tubercle of the humerus and into the capsule of the shoulder joint. Teres minor inserts into the lower facet of the greater tubercle of the humerus and into the capsule of the shoulder joint. Subscapularis inserts onto the lesser tubercle of the humerus.

Which of the following is not a direct branch of the axillary artery? A. anterior circumflex humeral B. posterior circumflex humeral C. thoracoacromial D. thoracodorsal E. subscapular

D. thoracodorsal The thoracodorsal and circumflex scapular arteries are branches of the subscapular artery, which comes directly off the third segment of the axillary artery. The anterior and posterior circumflex humeral arteries are also direct branches of the third segment of the axillary artery. The thoracoacromial artery is a branch of the second part of the axillary artery.

The victim of multiple shrapnel wounds to the upper limb must have his forearm amputated at midlength. Because of concomitant damage in the patient's arm, the surgeon must ligate the main artery at some point. The best chance of saving collateral circulation to the stump of the forearm would be when the ligature is placed just below which of the following? A. Beginning of brachial artery B. Origin of the deep brachial artery C. Origin of the superior ulnar collateral artery D. Origin of the inferior ulnar collateral artery E. Bifurcation of the brachial artery

E. Bifurcation of the brachial artery The brachial artery bifurcates near the elbow. It forms two branches that become the radial and ulnar arteries. If these arteries were ligated after this bifurcation, there would be a chance at saving collateral circulation to the forearm because the ulnar artery might have already given off its common interosseous branch, which could carry blood to the forearm through the anterior and posterior interosseus arteries. Ligating near the beginning of the brachial artery would stop blood from flowing through the rest of the upper limb. Ligating near the origin of the deep artery, by the origin of the superior ulnar collateral artery, or near the origin of the inferior ulnar collateral artery might preserve enough collateral circulation to supply the elbow. However, there would not be collateral circulation to the forearm. For a better picture of these arterial connections, see Netter Plate 434.

Upon finding a malignant tumor in the medial portion of the breast of a 40-year-old female, the surgeon began to search for the lymph nodes that would be the first ones reached by metastatic spread of cancer cells from this site. Which group(s) would have to be examined to determine whether metastasis had occurred? A. Central only B. Parasternal only C. Parasternal and apical D. Parasternal and lateral E. Parasternal and pectoral

E. Parasternal and pectoral As lymph drains from the breast, the majority of fluid travels to two groups of lymph nodes: the axillary and the parasternal. 75% of the lymph goes to the axillary lymph nodes, with the pectoral nodes being the first axillary nodes to receive the drainage. So, the pectoral nodes would need to be inspected to determine whether cancer had spread to the axillary system. Central and apical nodes are also part of the axillary system. These nodes might receive cancerous cells, but they are more distal sites of drainage. Cancer would be most likely to metastasize to the pectoral nodes first. The parasternal nodes receive most of the lymph that does not drain into the axillary nodes. They are an especially important route of drainage from the medial side of the breast. So, it is also important to survey these nodes to determine whether cancer has spread into the lymphatics.

The axillary nerve arises directly from which part of the brachial plexus? A. inferior trunk B. lateral cord C. medial cord D. middle trunk E. posterior cord F. superior cord

E. Posterior cord The axillary nerve and radial nerve are both terminal branches from the posterior cord. There are no nerves from the inferior trunk. The lateral cord gives off the musculocutaneous nerve and contributes a branch to the median nerve. The medial cord of the brachial plexus terminates with the ulnar nerve and a branch to the median nerve; it also gives off the medial pectoral nerve, the medial cutaneous nerve of the arm, and the medial cutaneous nerve of the forarm. The middle trunk of the brachial plexus has no direct branches. There is no superior cord of the brachial plexus--only a superior trunk!

A football player has a complete fracture of his radius just proximal to the insertion of the pronator teres muscle. As a result of the actions of the muscles attached to the proximal and distal fragments of the radius, which of the following combinations accurately reflects the orientation of the proximal and distal radial fragments? A. Proximal extended, and distal pronated B. Proximal extended and pronated, and distal supinated C. Proximal flexed, and distal pronated D. Proximal flexed, and distal supinated E. Proximal flexed and supinated, and distal pronated

E. Proximal flexed and supinated, and distal pronated The proximal fragment will be flexed and supi- nated by the biceps brachii and supinator muscles, while the distal fragment will be pronated by the action of the pronator teres and pronator quadratus muscles.

A person sustains a left brachial plexus injury in an auto accident. After initial recovery the following is observed: 1) the diaphragm functions normally, 2) there is no winging of the scapula, 3) abduction cannot be initiated, but if the arm is helped through the first 45 degrees of abduction, the patient can fully abduct the arm. From this amount of information and your knowledge of the formation of the brachial plexus where would you expect the injury to be: A. axillary nerve B. posterior cord C. roots of plexus D. superior trunk E. suprascapular nerve

E. Suprascapular nerve Let's take the observations one by one to break down this question. If the diaphragm is functioning normally, you know that the phrenic nerve is probably uninjured, which means that the C5 root has not been damaged. Since the scapula is not winged, there was no damage to the long thoracic nerve or the C5-7 nerve roots. Finally, since the patient cannot initiate abduction of the arm, you know that the suprascapular nerve is injured and supraspinatus has been denervated. But, the patient can abduct the arm once it is lifted to 45 degrees, so the deltoid muscle and the axillary nerve must be intact. Taking the answer choices one by one: The axillary nerve is ok, because deltoid is functioning. The posterior cord of the brachial plexus must also be intact, since this cord gives off the axillary nerve. The roots of the brachial plexus are ok, since the phrenic nerve and long thoracic nerve (which are derived from the roots) are still functioning. The superior trunk of the brachial plexus must also be undamaged, since this trunk contributes to the posterior cord which is intact. So, this means that the injury must be to the suprascapular nerve.

Examining an assembly line worker's complaint of tingling pain in her wrist with muscle weakness and atrophy, her physician diagnoses carpal tunnel syndrome. Which of the following muscles is most likely to be atrophied? A. Adductor pollicis B. Dorsal interossei C. Flexor digitorum superficialis D. Lumbricals 3 and 4 E. Thenar

E. Thenar The thenar muscles are located at the base of the thumb and are innervated by the median nerve (notably, by its recurrent branch), which passes through the carpal tunnel and is prone to injury in excessive repetitive movements at the wrist.

During a triathalon biking accident, a rider fell and landed with the handle bar of her bike forced upward into her right axilla. Subsequently while swimming in another triathalon event she found that her right arm tired so badly during the swimming portion that she could barely finish the event. During examination it was found that movements involving adduction, medial rotation and extension of her arm were particularly weak and affected her swimming stroke. The nerve injured was the: A. Accessory B. Dorsal scapular C. Lateral pectoral D. Medial pectoral E. Thoracodorsal

E. Thoracodorsal The thoracodorsal nerve innervates latissimus dorsi, which is an important muscle for adducting, medially rotating, and extending the arm. This is the muscle that is used when swimming the crawl. Since she can't do these motions, the triathelete must have injured her thoracodorsal nerve. Another indication of this injury is that the thoracodorsal nerve is particularly vulnerable following trauma to the axilla. The accessory nerve innervates trapezius. If this nerve was injured, the patient could not raise the tip of her shoulder. The dorsal scapular nerve innervates the rhomboids and levator scapulae. If this nerve was injured, the patient would have problems elevating or retracting her scapula. The lateral and medial pectoral nerves innervate pectoral major, a medial rotator and flexor of the arm. An injury to this nerve would cause a problem with flexion, not extension, of the arm.

Loss of function, paralysis, of which muscle would result in drooping or sagging of the shoulder? A. Erector spinae B. Latissumus dorsi C. Levator scapulae D. Rhomboideus major E. Trapezius

E. Trapezius The trapezius, innervated by the accessory nerve (CN XI), is the muscle responsible for elevating the tip of the shoulder. Erector spinae, which is innervated by the dorsal primary rami of C1-S5, extends and laterally bends the trunk, head and neck. Latissimus dorsi, innervated by the thoracodorsal nerve, allows the trunk to be lifted up to the arms (like with climbing or pull-ups). Levator scapulae, innervated by the dorsal scapular nerve, elevates the scapula. Rhomboidus major and minor are both innervated by the dorsal scapular nerve and aid trapezius in retracting the scapula.

A patient presents with numbness over his medial hand and atrophy of the hypothenar muscles after an injury several days ago over his medial humeral epicondyle. Which of the following nerves most likely was injured? A. Anterior interosseous B. Musculocutaneous C. Recurrent branch of median D. Superficial radial E. Ulnar

E. Ulnar The ulnar nerve is subcutaneous as it passes around the medial epicondyle of the humerus. In this location, it is vulnerable to compres- sion injury against the bone ("funny bone"), or entrapment in the cubital tunnel (beneath the ulnar collateral ligament).

A man suffers a penetrating wound through the anterior axillary fold, with resulting damage to one of the main terminal branches of the brachial plexus. Among the effects is a significant weakening of flexion of the elbow. One or more other effects to be expected is (are): A. Loss of cutaneous sensation on the tips of several fingers B. Only loss of cutaneous sensation on the anterolateral surface of the forearm C. Only weakening of flexion at the shoulder D. Weakening of flexion at the shoulder and loss of cutaneous sensation on the anterolateral surface of the arm E. Weakening of flexion at the shoulder and loss of cutaneous sensation on the anterolateral surface of the forearm

E. Weakening of flexion at the shoulder and loss of cutaneous sensation on the anterolateral surface of the forearm Flexion at the elbow is produced by biceps brachii and brachialis, and both of these muscles are innervated by the musculocutaneous nerve. So, you know that the musculocutaneous nerve was damaged. Beyond innervating the muscles that flex the forearm, the musculocutaneous nerve gives off the lateral antebrachial cutaneous nerve which provides sensory innervation to the anterolateral surface of the forearm. This means that the other symptom that would be present is a loss of cutaneous sensation on the anterolateral surface of the forearm. The biceps brachii and coracobrachialis muscles flex the arm, so you should have weakening of flexion at the shoulder - you would still have pectoralis major, a powerful arm flexor.

A knife cut results in a horizontal laceration to the thoracic wall extending across the midaxillary and anterior axillary lines just above the level of the T4 dermatome. Which of the following patient presentations will the emergency department physician most likely observe on examining the patient? A. Tingling along anterolateral forearm B. Supinated forearm C. Weakened elbow extension D. Weakened elbow flexion E. Winged scapula

E. Winged scapula This laceration probably severed the long thoracic nerve, which innervates the serratus anterior muscle. During muscle testing, the scapula will "wing" outwardly if this muscle is denervated.

A woman with breast cancer subsequently develops metastases in her vertebral column. The most direct route for spread of the tumor to the vertebral column was via: A. branches of the cephalic vein B. branches of the lateral thoracic vein C. branches of the thoracoacromial veins D. lymphatic vessels draining into the axilla E. branches of the intercostal veins

E. branches of the intercostal veins The most likely route for the cancer to reach the vertebral column is through the intercostal veins, i.e. hematogenous spread. The cephalic vein and thoracoacromial vein would not be draining the breast, and the lateral thoracic vein would not be directing blood toward the vertebral column. Lymphatic vessels may carry some tumor cells to the axillary lymph nodes and may participate in the spread of the cancer, but this isn't the best answer for this question. The most direct way for the cancer to spread to the vertebral column is through the venous system.

Interruption of cranial nerve XI would paralyze which muscle? A. deltoid B. latissimus dorsi C. levator scapulae D. rhomboideus major E. trapezius

E. trapezius Cranial nerve XI is another name for the accessory nerve, which innervates the trapezius. The deltoid muscle is innervated by the axillary nerve. (More to come with the upper limb.) Latissimus dorsi is innervated by the thoracodorsal nerve. Levator scapulae and rhomboidus major are both innervated by the dorsal scapular nerve, with the upper part of levator scapulae receiving some branches of C3 and C4.

Angina pectoris leads to referred pain, which radiates down the arm.

Referred pain from myocardial ischemia can be present along the medial aspect of the arm, usually on the left side, and is referred to this area by the medial cutaneous nerve of the arm (T1). The intercostal brachial nerve (T2) may also contribute.

Dislocation of the shoulder places this nerve in jeopardy of injury.

The axillary nerve (innervates the deltoid and teres minor muscles) can be injured by shoulder dislocations. This nerve passes through the quadrangular space before innervating its two muscles.

When asked to make a fist,the patient is unable to flex the first three fingers into the palm, and the fourth and fifth fingers are partially flexed at the MCP and DIP joints.

This suggests a lesion to the median nerve. The thenar muscles are affected, as are the long flexors of the digits (flexor digitorum superficialis). Unopposed extension of the first three fingers occurs, and absence of flexion at the PIP joints of fingers 4 and 5 is evident. The position of the hand is that of a "papal" or "benediction" sign.

A patient presents with a "claw hand" deformity.

Ulnar nerve "Claw hand" is a typical deformity of the ulnar nerve. The last two digits may be hyper- extended at the MCP joint (unopposed extensor digitorum-radial nerve innervated), flexed at the PIP joint (flexor digitorum superficialis-median nerve innervated) and extended at the DIP joint (loss of flexor digitorum profundus- ulnar nerve, and the action of the unopposed extensor expansion).

Despite injury to the radial nerve in the arm, a patient is still capable of supination of the forearm.

While the supinator muscle is denervated (loss of radial nerve), the biceps brachii muscle is innervated by the musculocutaneous nerve and is a powerful supinator when the elbow is flexed.


Set pelajaran terkait

MDR orgs like MRSA, susceptibilities, antibiotics, drug resistance, drug mechanisms

View Set

Information Research: Citation Formatting

View Set

Business Law 1 // Ch. 16 The Writing Requirement in Our Digital World

View Set

Lesson 4 homework endocrine system

View Set

The Spleen (Davies Board Review)

View Set

Managerial Accounting Homework Chapter 3

View Set

Art MO4 quiz ch 10, Art MO3, ART MO1, quiz 1, Ch.1-3, Art MO2 quiz 2( ch.4-7

View Set